Đến nội dung

hxthanh

hxthanh

Đăng ký: 30-10-2010
Offline Đăng nhập: Riêng tư
****-

#322631 Cách gõ công thức Toán trên diễn đàn

Gửi bởi hxthanh trong 05-06-2012 - 14:12

Ví dụ: :P

$$\begin{array}{|c|l|c|c|c|c|c|r|}
\hline
\text{TT}&\text{Đội}&\text{Trận}&\text{T}&\text{H}&\text{B}&\text{Điểm}&\text{HS}\\
\hline
1&\text{Gamma}&3&3&0&0&9&31\\
\hline
2&\text{Delta}&3&2&0&1&6&17\\
\hline
3&\text{Alpha}&3&1&0&2&3&-5\\

\hline
4&\text{Beta}&3&0&0&3&0&-42\\
\hline
\end{array}$$


#322622 Cách gõ công thức Toán trên diễn đàn

Gửi bởi hxthanh trong 05-06-2012 - 13:18

Đây là cách kẻ bảng bằng $\LaTeX$
(Dựa trên code của Thành :) )

$$\begin{array}{|c|c|cccc|cccc|}
\hline
1 & 2 & 3 & 4 & 5 & 6 & 7 & 8 & 9 & 0 \\
\hline
1 & 2 & 3 & 4 & 5 & 6 & 7 & 8 & 9 & 0 \\
\hline
1 & 2 & 3 & 4 & 5 & 6 & 7 & 8 & 9 & 0 \\
\hline
\end{array}$$

$$\begin{array}{|c|c|cccc|cccc|}
\hline
1 & 2 & 3 & 4 & 5 & 6 & 7 & 8 & 9 & 0 \\
\hline
1 & 2 & 3 & 4 & 5 & 6 & 7 & 8 & 9 & 0 \\
\hline
1 & 2 & 3 & 4 & 5 & 6 & 7 & 8 & 9 & 0 \\
\hline
\end{array}$$
Liền sau \begin{array} là {|c|c|cccc|cccc|} mô tả cách thức của bảng (theo các cột)
Dấu "gạch đứng" : "|" để ngăn cách cột
Mỗi chữ "c" mô tả độ rộng của cột tương ứng với một phần tử
\hline (headline) dòng kẻ trên
Mỗi hàng phải nhập đủ độ rộng của toàn bộ bảng (tổng số các chữ "c")
Mỗi phần tử ngăn cách nhau bởi dấu "và": &
Dấu \\ để xuống dòng
...
Hết!
Chúc mọi người thành công :D


#321429 Cho dãy A: 1, 2, 4, 6, 9, 12, 16, 20, 24, 28, 30, 35, 40, 45, 50, 54, ….. Tìm...

Gửi bởi hxthanh trong 01-06-2012 - 11:14

...(tiếp theo) ...
$\{a_n\}\;:\;1;\;2,4;\;6,9,12;\;16,20,24,28;\;30,35,40,45,50;\;54,...$
Diễn giải:
$\{a_n\}\;:\;$ (1 số là bội của 1); (2 số là bội của 2); (3 số là bội của 3); ...; (k số là bội của k); ((k+1) số là bội của (k+1)); ...
Trong đó: Số đầu tiên của nhóm (k+1) phải là số nhỏ nhất nhưng vẫn lớn hơn số cuối cùng của nhóm k

Lời giải cho dãy số này, ngắn nhất cũng mất 4 trang A4 (khi nào rảnh tôi post lên sau :D )

$\boxed{\text{ Đáp án: }}$

$a_n=b_n\left(n-\left\lfloor\dfrac{b_n(b_n+1)}{6}\right\rfloor\right);\;\;\;\text{với : }\;\;b_n=-\left\lfloor\dfrac{1-\sqrt{1+8n}}{2}\right\rfloor$

$b_n=-\left\lfloor\dfrac{1-\sqrt{1+8n}}{2}\right\rfloor$ chính là công thức xác định số hạng thứ $n$ của dãy số:

$\{b_n\}\;:\;1;\;2,2;\;3,3,3;\;4,4,4,4;\;...;k,k,...,k;\;...$
$\{b_n\}\;:\;$ (1 số 1); (2 số 2); (3 số 3); ... ; (k số k); ....


#321314 Cho dãy A: 1, 2, 4, 6, 9, 12, 16, 20, 24, 28, 30, 35, 40, 45, 50, 54, ….. Tìm...

Gửi bởi hxthanh trong 31-05-2012 - 20:49

Cho dãy A: 1, 2, 4, 6, 9, 12, 16, 20, 24, 28, 30, 35, 40, 45, 50, 54, …..
Tìm công thức tính số thứ i

1=1.1
2=2.1
4=2.2
6=3.2
9=3.3
12=3.4
16=4.4
20=4.5
24=4.6
28=4.7
30=5.6
35=5.7
40=5.8
45=5.9
50=5.10
54=6.9
...
Nhận thấy quy luật như sau:
$a_{n+1}>a_n\;(1)$
{1 số bội của 1, 2 số bội của 2, 3 số bội của 3, 4 số bội của 4, 5 số bội của 5, ... ,}
... (còn tiếp) ...


#321311 Cho dãy A: 1, 2, 4, 6, 9, 12, 16, 20, 24, 28, 30, 35, 40, 45, 50, 54, ….. Tìm...

Gửi bởi hxthanh trong 31-05-2012 - 20:34

Ta có :Công thức tính theo dãy truy hồi
$S_{2k}=S_{2k-1}+k$
$S_{2k+1}=S_{2k}+k$
( với kthuộc $N^{*}$ )
hay $S_{n}=S_{n-1}+\left [ \frac{n}{2} \right ]$

Không đúng ở số hạng thứ $10$ rồi em
$28=S_{10}=S_{2.5}=S_9+5=24+5\;\;?$


#320955 Chứng minh rằng dãy $(a_n)$ không tuần hoàn.

Gửi bởi hxthanh trong 30-05-2012 - 17:46

Cho dãy $\{a_n\};\;\left(n \geq 1\in \mathbb{N}^*\right)$ thỏa mãn $a_1+a_2+...+a_n$ là số nguyên lớn nhất không vượt quá $n\sqrt{2}$ . CMR dãy $\{a_n\}$ không tuần hoàn.

Giả sử dãy $\{a_n\}$ tuần hoàn với chu kỳ $t>0$, nghĩa là tồn tại số nguyên dương $t$ để cho:
$a_{n+t}=a_n,\;\;\forall n \ge 1\;\;(1)$
Khi đó, ta có:
$a_1+a_2+...+a_n+a_{n+1}+...+a_{n+t}=\left(a_1+a_2+...+a_n\right)+\left(a_1+a_2+...+a_t\right)=\left\lfloor\right(n+t)\sqrt 2\rfloor$

$\Rightarrow \left\lfloor n\sqrt 2\right\rfloor+\left\lfloor t\sqrt 2\right\rfloor=\left\lfloor\right(n+t)\sqrt 2\rfloor; \;\;\forall n \ge 1\;\;(2)$
Trong $(2)$ lần lượt cho $n=t, \;n=2t,\; ...,\; n=kt$ suy ra $k\left\lfloor t \sqrt 2\right\rfloor = \left\lfloor kt \sqrt 2\right\rfloor \;\;(*)$
Do $t \in \mathbb{N}^*$ nên $t \sqrt 2$ là số vô tỷ (Xin phép không chứng minh điều này)
Do đó ta đặt $t \sqrt 2 = \left\lfloor t \sqrt 2\right\rfloor + x;\;\;(0< x < 1)$
$(*)\Leftrightarrow k\left\lfloor t \sqrt 2\right\rfloor =\left\lfloor k\left\lfloor t \sqrt 2\right\rfloor +kx \right\rfloor \Leftrightarrow \left\lfloor kx \right\rfloor=0\;\;(**)$
Với $k$ lớn tùy ý thì rõ ràng $(**)$ không đúng
Mâu thuẫn $(**)$ dẫn tới điều phải chứng minh!
$\square$
___________________________________________________________________

$\boxed{\text{ Nhận xét: }}$ Để ý rằng dãy $\{a_n\}$ như trên được xác định bởi công thức:
$a_n=\left\lfloor n\sqrt 2 \right\rfloor - \left\lfloor (n-1)\sqrt 2 \right\rfloor$
Đây chính là dãy sai phân bậc nhất của dãy $\{\left\lfloor n\sqrt 2 \right\rfloor\}_1^\infty$
Việc chứng minh được dãy $\{a_n\}$ không tuần hoàn dẫn tới việc "hữu tỉ hóa" (tìm cách biểu diễn $\left\lfloor n\sqrt 2 \right\rfloor$ bằng phần nguyên một phân thức theo $n$) trở thành "bất khả thi"
Việc này không khác nào biến $\sqrt 2$ thành một phân số


#320780 Chọn nơi để tổ chức offline cho VMF hè năm nay :D

Gửi bởi hxthanh trong 30-05-2012 - 09:33

Hay là ta gửi $\boxed{\text{ Phiếu Mời }}$ cho từng mem (nằm trong danh sách) theo đường bưu điện?
  • Kir yêu thích


#320054 Cho một đa giác bất kì có tất cả các đường chéo bằng nhau. Hỏi đa giác đó có...

Gửi bởi hxthanh trong 27-05-2012 - 18:43

vô số cạnh ạ =). Cái này không khác gì đường tròn đâu!!. giả sử các đường chéo là đường kính thì cứ 2 điểm đối xứng bất kỳ thuộc đường tròn và 2 điểm khác như vậy ta luôn được điều như gt đặt ra... nói đại thế thôi nhưng mong là bạn hiểu được ý của mình :P

Thế nào là đường chéo của đa giác?
"Một đoạn thẳng nối hai đỉnh của đa giác mà không phải là cạnh của đa giác được gọi là một đường chéo của đa giác đó"
Vậy tất cả các đường chéo bằng nhau? Lục giác đều có bao nhiêu đường chéo?
- Đáp án đúng là Ngũ giác đều, với các đường chéo là một ngôi sao năm cánh.


#320022 Có tồn tại $m,n\in \mathbb{N}$ sao cho $\left [...

Gửi bởi hxthanh trong 27-05-2012 - 16:20

Bài toán: Hãy khẳng định hoặc phủ định mệnh đề sau:

Với mọi cặp số vô tỉ $s,r$ luôn tồn tại $m,n\in \mathbb{N}$ sao cho $\left[ {sm} \right] = \left[ {rn} \right]$

Câu trả lời là "mệnh đề sai"
- Thứ nhất nếu $rs<0$ ($r$ và $s$ trái dấu) thì không thể nào có được đẳng thức $\lfloor nr \rfloor = \lfloor ms\rfloor $ với mọi $m,n \in\mathbb{N}$
- Chắc tác giả muốn nói đến $rs>0$ (Nếu bằng $0$ thì đâu còn vô tỷ nữa :) )
- Câu khẳng định của đề bài là với mọi $r,s$ (dương) là không đúng
Chỉ cần $\dfrac{1}{r}+\dfrac{1}{s} = 1$ , theo định lý BEATTY ta có $\lfloor nr \rfloor \neq \lfloor ms\rfloor \;\;\forall m,n \in\mathbb{N}$

Xem phát biểu và chứng minh $\to$ định lý BEATTY


#320018 Ước lượng $\left\lfloor n\sqrt 2\right\rfloor...

Gửi bởi hxthanh trong 27-05-2012 - 16:08

$\boxed{\text{ BỔ ĐỀ }}$
ĐỊNH LÝ: BEATTY (Về phân hoạch tập số nguyên dương)
$\alpha;\;\beta$ là các số vô tỷ dương sao cho $\dfrac{1}{\alpha}+\dfrac{1}{\beta}=1$

Khi đó hai tập (hai dãy) vô hạn:
$\{A_n\}_1^\infty=\left\{ \lfloor \alpha \rfloor, \lfloor 2 \alpha \rfloor, \lfloor 3 \alpha \rfloor,... \right\}$ và $\{B_n\}_1^\infty=\left\{ \lfloor\beta\rfloor,\lfloor 2\beta\rfloor,\lfloor 3\beta\rfloor,... \right\}$
Lập thành 2 "phân hoạch" của tập các số nguyên dương $\mathbb{N}^*$
Nghĩa là:
$\{A_n\}_1^\infty \cap \{B_n\}_1^\infty= \varnothing$ và $\{A_n\}_1^\infty \cup \{B_n\}_1^\infty= \mathbb{N}^*$

Chứng minh:
Trước tiên ta chứng minh sự tách rời giữa $\{A_n\}_1^\infty$ và $\{B_n\}_1^\infty$
Thật vậy, giả sử $\exists i,j$ sao cho $A_i=B_j$, tức là $\lfloor i\alpha\rfloor = \lfloor j\beta \rfloor = k$
Khi đó, bởi vì $i\alpha$ và $j\beta$ đều là các số vô tỷ nên: $k<i\alpha<k+1$ và $k<j\beta<k+1$
hay: $\dfrac{i}{k+1}<\dfrac{1}{\alpha}<\dfrac{i}{k}$ và $\dfrac{j}{k+1}<\dfrac{1}{\beta}<\dfrac{j}{k}$

Cộng các bất đẳng thức này lại theo vế, ta được:

$\dfrac{i+j}{k+1}<\dfrac{1}{\alpha}+\dfrac{1}{\beta}=1<\dfrac{i+j}{k} \Rightarrow k<i+j<k+1 \Rightarrow \text{ vô lý! }$

Vậy ta có: $\{A_n\}_1^\infty \cap \{B_n\}_1^\infty= \varnothing$

Tiếp theo, ta sẽ chứng minh số tự nhiên $n$ bất kỳ sẽ phải có mặt hoặc trong $\{A_n\}_1^\infty$ hoặc trong $\{B_n\}_1^\infty$
Thật vậy, cũng bằng phương pháp phản chứng, ta giả sử $n$ không xuất hiện trong cả hai dãy trên, khi đó:
$\exists i,j$ sao cho: $\begin{cases} i\alpha<n \\ (i+1)\alpha > n+1 \end{cases}$ và $\begin{cases} j\beta<n \\ (j+1)\beta > n+1 \end{cases}$

Hay là: $\dfrac{i}{n}<\dfrac{1}{\alpha}<\dfrac{i+1}{n+1}$ và $\dfrac{j}{n}<\dfrac{1}{\beta}<\dfrac{j+1}{n+1}$

Cộng các bất đẳng thức này lại theo vế, ta được:

$\dfrac{i+j}{n}<\dfrac{1}{\alpha}+\dfrac{1}{\beta}=1<\dfrac{i+j+2}{n+1} \Rightarrow i+j<n<i+j+1 \Rightarrow \text{ vô lý! }$

Như vậy ta có: $\{A_n\}_1^\infty \cup \{B_n\}_1^\infty= \mathbb{N}^* $

$\square$ Định lý được chứng minh xong!
__________________________________________________________________________________

Áp dụng bổ đề cho bài toán trên:

Do $(2+\sqrt 2)$ và $\sqrt 2$ đều là các số vô tỷ, mà ta có: $\dfrac{1}{2+\sqrt 2} + \dfrac{1}{\sqrt 2} = 1$

nên hai dãy $\{ \lfloor n(\sqrt 2 +2) \rfloor \}_1^\infty $ và $\{ \lfloor m \sqrt 2 \rfloor \}_1^\infty $ là phân hoạch của tập $\mathbb{N}^*$

Suy ra $\lfloor n(\sqrt 2 +2) \rfloor \neq \lfloor m(\sqrt 2) \rfloor \Leftrightarrow \lfloor n \sqrt 2 \rfloor + 2n \neq \lfloor m \sqrt 2 \rfloor $

$\square$


#319941 Tìm Min và Max của $$P=[x[x]]+[y[y]]$$

Gửi bởi hxthanh trong 27-05-2012 - 02:24

Cho $x,y$ là các số thực âm thoã mãn$(x-\frac{1}{2})^2+(y-\frac{1}{2})^2\le \frac{27}{5}\;\;\; (1)$.
Tìm Min và Max của $$P=[x[x]]+[y[y]]$$

Tôi vẫn chưa hiểu vì sao bài này lại cho số lẻ vậy!
Do $x,y<0$ nên ta có:
$\left\lfloor x \right\rfloor \le -1 \Rightarrow x \left\lfloor x \right\rfloor >0 \Rightarrow \left\lfloor x \left\lfloor x \right\rfloor \right\rfloor \ge 0$
Do đó:
$P=\left\lfloor x \left\lfloor x \right\rfloor\right\rfloor+ \left\lfloor y \left\lfloor y \right\rfloor \right\rfloor \ge 0$
Chọn $x=y=-\dfrac{1}{2}$, $($thỏa mãn $(1))$
Suy ra $P=0$
Vậy $\min(P)=0$

Biến đổi $(1)$
$(1) \Leftrightarrow x^2-x+y^2-y \le \dfrac{49}{10} = 4,9$

Ta sẽ chứng minh rằng: Với $x<0$ thì $\left\lfloor x \left\lfloor x \right\rfloor\right\rfloor \le x^2-x\;\;(*)$
Thật vậy, ta có: $\left\lfloor x \right\rfloor+\left\lfloor -x \right\rfloor = \begin{cases} 0 & \text{ với }x\in \mathbb{Z} \\ -1 & \text{ với }x\not\in\mathbb{Z}\end{cases}$
$ \Rightarrow \left\lfloor x \right\rfloor+\left\lfloor -x \right\rfloor \ge -1$
$ \Rightarrow \left\lfloor x \right\rfloor \ge - \left\lfloor -x \right\rfloor -1 \ge x -1$
$ \Rightarrow x \left\lfloor x \right\rfloor \le x^2-x$ (Do $x<0$)
Từ đó ta chứng minh được $(*)$
Như vậy áp dụng $(*)$ ta được:
$P = \left\lfloor x \left\lfloor x \right\rfloor\right\rfloor+ \left\lfloor y \left\lfloor y \right\rfloor \right\rfloor \le x^2-x+y^2-y \le \dfrac{49}{10} = 4,9$
Suy ra, ta sẽ có $\max(P)=4$ nếu ta chỉ ra được một cặp giá trị $(x,y)$ phù hợp.
Chẳng hạn chọn $(x,y)=\left(-\dfrac{11}{10},-\dfrac{11}{10}\right)$

Kết luận:
$\boxed{\min(P)=0;\;\;\max(P)=4}$


#319830 Chứng minh định lý Lớn Fermat với kiến thức PT

Gửi bởi hxthanh trong 26-05-2012 - 20:37

Tác giả phân tích rất hay, rất tiếc đã ngộ nhận ở một điểm sau:

... Từ đó nhận thấy $(x,y,z)$ là tập hợp những tam giác có số đo $3$ cạnh là số tự nhiên, trong đó $z$ là cạnh dài nhất $\Rightarrow$ góc nhìn $z:\;\;60^\circ< \alpha < 180^\circ$
Định lý cosin: $z^2=x^2+y^2-2xy\cos\alpha$
Đối với $\alpha \neq 90^\circ, \;120^\circ$ thì $\cos\alpha$ không nguyên
$\Rightarrow z \not\in \mathbb{N},$ pt $(1)$ không có nghiệm tự nhiên khi $n>1$
...

Với $60^\circ< \alpha < 180^\circ$ thì $-1 < \cos\alpha <\dfrac{1}{2}$
Do vậy nó có thể bằng $\cos\alpha=\dfrac{1}{3};\;\dfrac{1}{4};...$
Ví dụ: $\cos\alpha=\dfrac{1}{3}$ thì chỉ cần $x$ hoặc $y$ chia hết cho $3$, số còn lại thích hợp lúc đó $z^2$ sẽ nguyên và rất có thể $z$ sẽ nguyên.
Do vậy không thể vội vàng kết luận như phần trên (bôi đỏ)


#319737 Tính tổng quát giá trị biểu thức: A=$1+\frac{1}{2}+\frac{1}{3}...

Gửi bởi hxthanh trong 26-05-2012 - 14:32

Quy luật thì ... cũng có nhưng chẳng ích gì :P
$\sum_{i=1}^n i^k=P_{k+1}(n)$
- $P_{k+1}(n)$ là một đa thức bậc $k+1$
- Luôn có 2 nghiệm là $x_1=0$ và $x_2=-1$
- Nếu $k$ chẵn thì có thêm nghiệm $x_3=-\dfrac{1}{2}$


#319680 Tính tổng quát giá trị biểu thức: A=$1+\frac{1}{2}+\frac{1}{3}...

Gửi bởi hxthanh trong 26-05-2012 - 10:46

Đúng rồi em ạ
$\mathcal{H}^{k}_n = \sum_{i=1}^n \dfrac{1}{i^k}$ là một hàm Hyperbol điều hòa (không sơ cấp chút nào!)

Vì vậy không cần phải cố gắng tìm công thức sơ cấp đâu các em ạ!


#319641 Tìm $1000$ chữ số cuối của số : $$N=1+50+50^2+...+50^{99...

Gửi bởi hxthanh trong 26-05-2012 - 08:37

Theo tôi thì đi tìm chữ số hàng thứ 1000 còn ý nghĩa hơn là đi ngồi viết cái kết quả của quy luật tuần hoàn kia!